theory sem II

Ace your homework & exams now with Quizwiz!

Prior intracranial hemorrhage.

A client presents to the er with a myocardial infarction. Prior to administering a prescribed thrombolytic agent, the nurse must determine whether the client has which absolute contraindiction to thrombolytic therapy.

Paresthesia

A client presents to the er, stating numbness and tingling occurring down the left leg into the left foot when documenting the experience, which medical terminology would the nurse be most correct to report.

- pain in the buttocks that extends down the leg. - Weakness in the muscles of the left leg.

A client reports beginning to experience lower back pain after lifting a heavy box which symptoms would lead the provider to suspect lumbar nerve compression. Select all that apply.

Ventricle depolarization

A client was second degree atrioventricular heart block is admitted to the coronary care unit, the nurse closely monitors the client's heart rate and rhythm when interpreting the clients electrocardiogram strip the nurse knows that the QRS complex represents.

Stomatitis

A client with a brain tumor is undergoing radiation and chemotherapy for treatment of cancer, the client has recently reported swelling in the gums, tongue and lips, which is most likely cause of these symptoms.

The result of Venus bleeding into the space below the dura.

A client's family is trying to understand the clients diagnosis of acute subdural hematoma, the nurse would best explain the condition by stating that a subdermal hematoma is.

Apply a bleach solution prior to sweeping or vacuuming the floor.

A family will be staying in a cabin by a lake, and upon arriving observed throat and droppings on the floor in the kitchen, what is the best way for the family to clean up in order to avoid contracting hantavirus from the feces.

Clubbing of the fingers.

A finding by the nurse may indicate that the patient has chronic hypoxia.

Cerebrospinal fluid leak (CSF)

A halo sign is indicative of which of the following complications of brain injury.

Notify the client prior to touching the client.

A legally blind client is in pre op area prior to appendectomy. What steps does the nurse take to effectively communicate with this client?

Foul smelling discharge from the penis.

A male patient comes to the clinic and is diagnosed with gonorrhea, which symptom most likely prompt him to seek medical attention.

Decreases resting heart rate.

A nurse is caring for a client who experienced an mi. The client is ordered to receive motoprolol. The nurse understands that this medication has which therapeutic effect.

Observed for muscle twitching and numbness or tingling of the lips, fingers and toes.

A nurse is caring for a client who had a thyroidectomy and is at risk for hypocalcemia. What should the nurse do?

A slow rate of growth.

A nurse is caring for a client with a benign breast tumor. The tumor has. Which characteristic.

Fungi

A nurse is caring for a client with ringworm. Which of the following microorganisms causes ringworm in a client.

Maintain and improve cerebral tissue perfusion.

A nurse is caring for a patient diagnosed with a hemorrhagic stroke. What goal is a priority for this patient?

Erosion of the lining of the stomach or intestine.

A nurse is caring for a patient who just has been diagnosed with a peptic ulcer,when teaching the patient about his new diagnosis. How should the nurse best describe a peptic ulcer?

asterixis

A nurse is caring for a patient with hepatic and encephalopathy While making the initial shift assessment, the nurse notes that the patient has a flapping tremor of the hands, the nurse document. The presence of what sign of liver disease.

women

A nurse is planning the assessment of a patient who is exhibiting signs and symptoms of an autoimmune disorder, the nurse should be aware that the incidence and prevalence of autoimmune diseases is known to be higher. Among what group?

Hepatitis c increases a person's risk for liver cancer.

A nurse is preparing a presentation for a local community group about hepatitis which of the following with the nurse include.

- loss of lens accommodative power. - Development of lens opacities. - Loss of eyelid skin elasticity.

A nurse is preparing a presentation for a local senior citizens group about changes in the eyes that accompany age which of the following would the nurse most likely include select all.

Speech

A patient has been diagnosed with damage to Broca's area of the left frontal lobe to document the extent of damage the nurse would assess the patients:

Hospice care.

A patient has been diagnosed with pancreatic cancer and has been admitted for care. Following initial treatment, the nurse should be aware that the patient is most likely to require. Which of the following.

Heart transplant

A patient has been living with dilated cardiomyopathy for several years but has experienced worsening symptoms despite aggressive medical management. The nurse anticipates what potential treatment?

" if I get chest pain or shortness of breath, it will show up on my telemetry monitor."

A patient has been placed on telemetry following treatment for a non St. wave elevation myocardial infarction (NSTEMI). Which of the patients following statements indicates that the nurse should perform further patient teaching?

Instruct the patient on the side effects of flushing, throbbing headache and tachycardia.

A patient has been recently placed on nitroglycerin, which of the following should be included in the patient teaching plan.

Type one diabetes.

A 15 year old child is brought to the er with symptoms of hyperglycemia and is subsequently diagnosed with diabetes based on the fact that the child's pancreatic beta cells are being destroyed, the patient would be diagnosed with what type of diabetes.

hemoptysis

A 22 year old college student recent return from a backpacking trip to Southeast Asia and has been experiencing increasing in Malaise over the past 2 weeks today he is seeking care because he reports that he coughed up blood during a particularly heavy coughing fit this morning. The nurse should document the presence of

women often present with an mi much differently than men do.

A 60 year old woman has been brought to the er by ambulance after she experienced a sudden onset of dyspnea and phoned 911, the woman is obese but claims an unremarkable medical history and denies chest pain. When assessing this patient. The nurses in the er should be aware that.

Pnemonia

A 62 year old female is 2 weeks,CABG & is returning to her cardiologist due to new symptoms. She reports heaviness in her chest, and pain between her breast. She reports that leaning forward decreases the pain. After his thorough examination, the cardiologist admit her to the hospital to rule out pericarditis. Which of the following is not a contributing cause to pericarditis?

Superinfection

A 70 year old client with chronic obstructive pulmonary disease has a respiratory infection being treated with antibiotics, he is also taking oral corticosteroids to assist in decreasing the inflammation in the lungs the client is prone to.

type O

A client comes to the er and is bleeding profusely. Blood transfusion is ordered on the client is being typed in cross matched while awaiting the results, the physician orders a unit of packed red blood cells to administer a stat. What blood type would the nurse expect to be used?

elevated ST segment

A client comes to the er. Complaining of chest pain and electrocardiogram reveals myocardial ischemia and an anterior wall. Myocardial infarction. Which ECG characteristics does the nurse expect to see?

Explain the location of items using clocks cues

A patient has had a sudden loss of vision after head trauma. How should the nurse best describe the placement of items on the dinner tray?

Apply a skin barrier to the peristomal skin prior to applying the pouch.

A patient has had an ileostomy. Created for the treatment of irritable bowel disease and the patient is now preparing for discharge, what should the patient be taught about changing this device in the home setting?

rheumatic fever

A client goes to the clinic reporting fever, chills sore throat and diagnosed with streptococcal pharyngitis, a nurse knows that early diagnosis and effective treatment is essential to avoid which preventable disease?

Chronic traumatic encephalopathy

A client had a long and successful ice hockey career but has been forced to retire due to symptoms of depression, memory loss and difficulty with gait and balance, the neurologist believes the most likely cause of these symptoms is.

Take prophylactic antibiotics.

A client has a history of rheumatic fever as a child. Which instruction should be provided before the client? Has any dental work done?

Replace fluid and electrolytes.

A client has a physician's order for NPO. Nothing by mouth following abdominal surgery to repair a bowel obstruction, the client has a nasal gastric tube inserted to low, intermittent suction. The client requires intravenous therapy to:

5040

A client has a stroke volume of 70 ml and a heart rate of 72 beats per minute. The nurse evaluates that the cardiac output is?

All options are correct.

A client has been a client in the ICU for 17 days. Seriously ill. A multitude of medications are ordered for the clients treatment regime, most of which are administered intravenously what is an indication for the use of IV medications? Select all that apply.

-slurred speech -vomiting -weakness on one side of the body

A client has been diagnosed with a concussion and is preparing for discharge from the er. The nurse teaches the family members who will be caring for the client to contact the physician or return to the er if the client demonstrates or reports which complications select all that apply.

Alcoholic beverages

A client has been having cluster headaches intermittently over the last year. In an effort to determine the trigger for the cluster headaches, the client has maintained a journal of all oral consumption what on the list would the nurse suspect could be triggering headaches.

flaccidity

A client has been in a coma since being in a motor vehicle accident, emergency surgery relieved intracranial pressure, but to date the client has not regained consciousness. Which motor response is indicative of the most serious condition?

an old MI

A client has had a 12 lead. ECG completed as part of an annual physical examination, the nurse notes an abnormal Q wave on an otherwise unremarkable ECG. The nurse recognizes that this finding indicates:

NSAIDS

A patient has sought care because of recent dark colored stools as a result, a fecal occult blood test has been ordered, the nurse should instruct the patient to avoid which of the following prior to collecting a stool sample.

potassium

A patient in the recovery room after cardiac surgery begins to have extremity paresthesia peaked, t waves and mental confusion. What type of electrolyte imbalance does the nurse suspect this patient is having?

Measles

A client in the emergency department is diagnosed with a communicable disease when complications of the disease are discovered, the client is admitted to the hospital and placed in respiratory isolation, which infection warrants airborne isolation.

nausea

A client is admitted for evaluation of cerebral aneurysm. Which assessment finding is of greatest importance in prioritizing nursing care to this client.

Right, lower quadrant.

A client is admitted with a diagnosis of acute appendicitis when assessing the abdomen the nurse would expect to find rebound. Tenderness at which location.

Transient ischemic attack.

A client is hospitalized when presenting to the er with right sided weakness within 6 hours of being admitted, the neurologic deficits had resolved in the client was back to his pre symptomatic state. The nurse caring for the client knows that the probable cause of the neurologic deficit was what.

Cereal, soy beans and spinach.

A client is prescribed. Warfarin client teaching has included instructions to avoid a diet rich in foods that contain vitamin k what sources of food. Should the nurse instructs the client to avoid.

Moon face

A client is receiving long term treatment with high dose corticosteroids, which of the following with the nurse expect the client to exhibit.

I guess the doctor could not remove the entire tumor.

A client is recovering from a craniotomy, a tumor debulking what comment by the client indicates to the nurse a correct understanding of what the surgery entailed.

A small portion of the anterior capsule is removed, ultrasound is emitted through a probe and suction is used to extract the lens.

A client is scheduled to undergo surgery to remove a cataract and the left eye using phacoemulsification. What phacoemulsification is used,a:

vascular occlusion in small vessels, decreasing blood and oxygen to the tissues.

A client is seen in the emergency with severe pain related to sickle cell crisis. What does the nurse understand is occurring with the client.

Noncontrast computed tomography.

A client is suspected of having a stroke, which is the initial diagnostic test for a stroke.

-temperature of 102°F -heart rate of 120 beats/min -respiratory rate of 24 beats per min

A client is suspected of sepsis from a post surgical incision infection. What characteristic of substance would the nurse recognized select all that apply.

- a visual field of floating particles. - Definite area of blank vision. - Momentary flashes of light.

A patient is being seen and the ophthalmology clinic for a suspected detached retina. What clinical manifestations does the nurse recognize as significant for a retinal detachment? Select all that apply.

I have to keep your door shut at all times, I'll open the curtains so that you don't feel so closed in.

A patient on airborne precautions ask the nurse to leave the door open. What is the nurses best reply?

Standard and airborne precautions.

A patient on your unit is found to have pulmonary tuberculosis. TB what is the most appropriate precaution for the staff to take to prevent transmission of TB?

Cerebral aneurysm

A patient presents to the er. With complaints of having an exploding headache for the last 2 hours, the patient is immediately seen by a triage nurse who suspects the patient is experiencing a stroke, which of the following is a possible cause based on the characteristic symptom.

The patients polyps constitute a risk factor for cancer.

A patient screening colonoscopy revealed the presence of numerous polyps in the large bowel. What principles should guide the subsequent treatment of this patient's health problem?

Elevated protein levels in the CSF.

A patient suspected of having Guillain-barre syndrome has had a lumbar puncture for cerebral spinal fluid CSF evaluation. When reviewing the laboratory results. What does the nurse find that is diagnostic for this disease.

Frontal lobe

A patient sustained a head injury during a fall and has changes in personality and effect. What part of the brain does the nurse recognize has been affected in this injury.

The patient had a vagal response.

A patient tells the nurse "I was straining to have a bowel movement and felt like I was going to faint, I took my pulse and it was so slow." What does the nurse understand occurred with this patient?

-diuretics -digoxin -inotropic agents

A patient who had CABG is exhibiting signs of cardiac failure. Which medication does the nurse anticipate administering for this patient? Select all that applies.

Excisional biopsy

A patient will be having an endoscopic procedure with a diagnostic. Biopsy. What type of biopsy does the nurse explain will remove an entire piece of suspicious tissue?

Check the patients indwelling urinary catheter for kinks to ensure patency.

A patient with a c 5 spinal cord injury is tetraplegic. After being moved out of the ICU, the patient complains of a severe throbbing headache. What should the nurse do first?

The patient is particularly susceptible to infection.

A patient with a diagnosis of HIV exhibits a decreased level of t lymphocytes. What consequence does this state present for this patient?

Vitamin k

A patient with a history of a-fib has contacted the clinic, saying that she is accidentally overdosed on her prescribed coumadin. The nurse recognize the possible need for what antidote.

Immunosuppression.

A patient with a history of dermatitis takes corticosteroids on a regular basis. The nurse assess the patient for which of the following complications of therapy.

The patient has a rigid "board like" abdomen that is tender.

A patient with a history of peptic ulcer disease has presented to the er in distress, what assessment finding would lead the er nurse to suspect that the patient has a perforated ulcer.

amoxicillin (amoxil)

A patient with history of rheumatic heart disease knows that she is at risk for bacterial endocarditis when undergoing invasive procedures. Prior to a scheduled cystoscopy, the nurse should ensure that the patient knows the importance of taking which of the following drugs.

The patient should be monitored for bone marrow depression.

A patient with metastic cancer has developed trigeminal neuralgia and is taking carbamezpine (tegretol) for pain relief. What principle applies to the administration of this medication?

Testosterone

A teenage male client has not developed facial hair, a deeper voice or other secondary sex characteristics. What hormone would the nurse expect would be checked for this client.

electroencephalography (EEG)

A trauma patient in the ICU has been declared brain dead. What diagnostic test is used in making the determination of brain death?

Clients eating and sleeping habits.

A visiting nurses setting up an insulin schedule for an older adult who has diabetes mellitus. What should the nurse consider when determining the dose time?

Goiter

A woman with a progressively enlarging neck comes into the clinic she mentions that she has been in a foreign country for the previous 3 months and that she didn't eat much while she was there because she didn't like the food, she also mentions that she becomes dizzy when lifting her arms to do normal household chores when dressing a endocrine disorder should the nurse expect the physician to diagnose?

Ulcerative colitis

A client presents to the emergency department with complaints of acute gi distress, bloody diarrhea, weight loss and fever, which condition in the family history is most pertinent to the client's current health problem.

gamma globulins

a nurse is providing care to a cancer client which protein in plasma functions primarily as immunologic agents ?

Increased thirst, hunger and urination.

a nurse is providing education to a client who is newly diagnosed with diabetes. What are classic symptoms associated with diabetes?

Always carry a form of fast acting sugar.

a nurse is working for the summer at a camp of adolescents with diabetes when providing information on the prevention and management of hypoglycemia. Which action should the nurse promote?

Protamine sulfate

a post operative client is receiving heparin after developing thrombophlebitis. The nurse monitors the client carefully for bleeding and other adverse effects of heparin. If the client starts to exhibit signs of excessive bleeding the nurse should expect to administer an antidote that is specific to have heparin.Which agent fits out description?

First thing in the morning.

a sputum study has been ordered for a patient who has developed course, chest crackles and a fever. At what time should the nurse best collect the sample?

At least 18 -24 inches above infusion site.

leaving your surgical site post op Martin Franks has hypertonic solution, running via gravity at a 100 ml per hour to decrease his post-op edema before transporting to his room, the IV pole is set at the proper height to overcome the pressure within Martin's veins. What is the heights?

Constrict pupil

miotic solutions are often ordered in the treatment of glaucoma. Which is the best nursing rational for the use of this medication.

Direct sunlight

sulfonamides can cause photosensitivity. Which of the following should be avoided by patients taking a sulfonamide medication.

unstable

when a client who has been diagnosed with angina pectoris reports experiencing chest pain more frequently even at rest, that the period pain is longer and that it takes less stress for the pain to occur the nurse recognizes that the client is describing which type of angina?

Lipase

which enzyme aids in the digestion of fats.

injection of filgrastim (neupogen)

which is the best option for raising the white blood cell count in a cancer client. Who is at risk for congestive failure.

Restrictive cardiomyopathy. (RCM)

which type of cardiomyopathy are characterized by diastolic dysfunction caused by rigged ventricular walls that impair diastolic filling and ventricular stretch?

wheezes

while auscultating the lungs of a client with asthma than nurse, here's a continuous, high pitched whistling sound on expiration. The nurse will document the sound as which of the following.

administer the digoxin

Before administering digoxin to a client with valvular disease the nurse assesses the apical heart rate as 62 beats per minute, the client's usual rate ranges between 66 to 72 beats per minute, which is the best action for the nurse to take?

Thymus gland

Chance Opferman a 16 year old high school sophomore is being seen by a pediatric endocrinologist within the group where you practice nursing as an alteration in the production of t lymphocytes and is undergoing diagnostics. What gland aids in the development of t lymphocytes?

Seasonal changes

Karen Nichols, a 32 year old concert promoter, has a 12 year history of migraine headaches. As she discusses her most recent headaches, she voices frustration over how her life is at the mercy of these headaches as she describes the characteristics of her head pain, you begin to discuss a potential triggers of her migraines. Which of the following is not a potential trigger to migraines.

Ammonia.

Lactulose (cephulac) is administered to a patient diagnosed with hepatic encephalopathy to reduce which of the following.

one-third

Lillian Anderson, a 73 year old retired dancer, is being seen by a neurologist in the group where you practice nursing, she reports lightheadedness speech disturbance and left sided weakness, which last for several hours the neurologist diagnosed a transient ischemic attack which caused miss Anderson great concern during your client education with miss Anderson. You discuss the frequency of tias developing into stroke. How many people who suffer Tia's developed CVA?

Pituitary disorder

Lydia kranston a 3 year old female is being seen by a physician in the endocrinology group where you practice nursing, she is a significant height deficit and is being seen for diagnostic purposes. What could be the cause of her disorder?

Thrombolytic therapy

Medical management of a client with a cerebral aneurysm does not include.

The stage 3 pressure ulcer on the left heel.

Nurses caring for a client receiving chemotherapy, which assessment finding places, the client at great risk for infection.

Bradycardia and hypertension.

Nurses caring for a patient with autonomic dysreflexia. What clinical manifestations would the nurse expect in this patient.

Rising blood pressure and bradycardia.

On caring for a client with a head injury, a nurse must stay alert for signs and symptoms of increased intracranial pressure ICP, which cardiovascular findings are late indicators of increased ICP.

Change in mental status.

On caring for a client with cirrhosis which symptoms should the nurse report immediately?

Respirations, 10 breaths per minute.

On managing the post operative pain after a pneumonectomy, the nurses most concerned about which assessment data.

mannitol

Osmotic diuretics are an essential intervention for reducing cerebral edema. Which of the following drugs is most frequently prescribed for this situation.

-maintaining Mr. Cavanaghs airway -promoting adequate caloric and fluid intake

Paul Cavanagh, pay 63 year old retired teacher had oral cancer and had extensive surgery to excise the malignancy. during his recovery while in he hospital where you practice nursing, you have specific goals for your nursing management of the patient and his condition, which of the following interventions are the focus of your attention? Choose all correct options.

- onset of stroke within 3 hours of initiation of therapy. - Normal prothrombin (PT) and partial thromboplastini (PTT) times

Pay nurses administering intravenous tissue plasminogen activator (tPA) to a client having a stroke, which criteria must be met before administering the agent? Select all that apply.

Difficulty breathing

Phil Smith, 93 year old male, is having a pulmonary angiography performed in the radiology department where you practice nursing. Which sign would indicate a allergic reaction to the contrast medium.

15

The Glasgow Coma Scale is a common screening tool used for patients with a head injury during the physical exam, the nurse documents that the patient is able to spontaneously open her eyes, obey verbal commands and is oriented. The nurse records the highest score of:

cerebellum

The brain is a complex structure and is divided into 3 parts, the cerebrum, the cerebellum and the brainstem, the cerebrum is divided into 2 hemispheres and is further divided into 4 lobes per hemisphere, which section of the brain controls and coordinates muscle movements.

Autonomic dysreflexia.

The client has been brought to the er. By their caregiver, the caregiver says that she found the client diaphoretic, nauseated, flush and complaining of a pounding headache when she came on shift. What are these symptoms indicative of?

Employs the z track technique.

The client is a young thin woman who is prescribed iron dextran IM. The nurse when admisinstering the medication,

Halo sign

The nurse is caring for a client with a head injury after a fall from a hay loft, which of the following indicates the presence of or leaking of cerebral spinal fluid.

Thrombocytopenia

The nurse is observing the skin of a client who is taking medications that depress the hematopoetic system and notices multiple areas of ecchymosis on the arms, bleeding for a prolonged period after an iV was started and reports of black tarry stool. What does the nurse understand may be a side effect of the medication that the client displays.

They have the beginning of a pressure sore.

The nurse receives a call from the caregiver of a client with a spinal cord injury. The caregiver informs you that the client has a red and macerated area at the base of the sacrum what should the nurse suspects with the client.

Abdominal paracentesis

The nurses assisting the physician with a procedure to remove ascitic fluid from a client with cirrhosis what procedure does the nurse ensure the client understands will be performed.

Cardiac muscle inflammation

The nurses caring for a patient diagnosed with myocarditis. The nurse understands that, which of the following is the hallmark of myocarditis?

- rate of growth - Ability to cause death. - Ability to spread..

The nurses describing some of the major characteristics of cancer to a patient who has recently received a diagnosis of malignant melanoma when differentiating between benign and malignant cancer cells the nurse should explain differences in which of the following aspects? Select all that apply.

- gently press a dry, sterile gauze square over the site. - Remove the venipuncture device by pulling it out without hesitation following the course of the vein. - Cover the site with a dressing or bandage.

The nurses discontinuing an intravenous infusion. After washing the hands, the nurse clamps the tubing, remove the tape, holding the depressing and venipuncture device in place and put clean gloves on. Place in order the remaining steps of the procedure that the nurse would follow.

At bed time.

The nurses instructing the client on use of ophthalmic eye ointment for treatment of an eye disorder. The ointment is ordered one daily. When is the best time to apply the aliment?

Conductive.

There are several types of hearing loss, which type of hearing loss benefits most from the use of a hearing aid.

Visual acuity.

This corner is testing the kindergarten class with a Snellen chart. What is the nurse testing the children for?

Atropine

Treatment of sympathetic bradycardia includes which of the following.

infiltration

You are making initial shift assessments on your patients. While assessing one patients peripheral IV site, you know edema around the insertion site. How should you document this complication related to IV therapy?

Extravasation of the medication.

You are the nurse caring for a patient who is to receive IV daunorubicin, a chemotherapeutic agent. He has started the infusion and check the insertion site as per protocol during your most recent check, you know that the IV has infiltrated so you stop the infusion. What is your main concern with this infiltration?

Sitting on the edge of the bed.

In which position should the client be placed for thoracentesis

hypertrophic

In which type of cardiomyopathy does the heart muscle actually increase in size and mass weight, especially along the septum?

Left ventricle to left atrium.

Incomplete closure of the mitral valve results in backflow of blood from the:

Bright flashing lights.

Which is to be an accurate clinical manifestation of a retinal detachment.

Monitoring the patency of an indwelling urinary catheter.

Which nursing intervention can prevent a client from experiencing autonomic dysreflexia?

Stroke volume

Which term describes the amount of blood ejected per heartbeat.

Ataxia

Which term refers to the inability to coordinate muscle movements resulting in difficulty walking?

presbycusis

Which term refers to the progressive hearing loss associated with aging.

Acute angle closure glaucoma.

Which type of glaucoma presents an ocular emergency?

Pallative

Which type of surgery is used in attempt to relieve complications of cancer?

Notify the physician immediately.

While assessing a client with coronary artery disease following a coronary arteriography, the nurse observes that the client has cool toes, and absent distal peripheral pulse, and a rapid pulse. Which intervention would be most appropriate?

Left ventricle

Within the physiology of the heart, each chamber has a particular role in maintaining cellular oxygenation. Which chamber of the heart is responsible for pumping blood to all the cells and tissues of the body?

Right ventricle

Within the physiology of the heart, each chamber has a particular role in maintaining cellular oxygenation. Which chamber of the heart is responsible for pumping blood to the lungs to be oxygenated.?

The client should be placed in a private room in possible

. When a hospitalized client requires contact precautions, what response is necessary?

The duration of oxygen deprivation to the patient's cardiac cells.

56 year old man has been brought to the er by emergency medical services and has been diagnosed with myocardial infarction, based on his presentation and electrocardiogram. The patient has been identified as a candidate for percutaneous transluminal coronary angioplasty. PTCA, the nurse who is providing care for this patient, should recognize that the extent of the cardiac damage will primarily depend on.

High LDL level

62 year old stock broker attend his annual physical appointment, indicates physical changes since his last examination, he reports chest pain and palpulation during and after his morning jobs, Jack's family history reveals a coronary artery disease. His lipid profile reveals his LDL level is to be 122 mg/dl. Which of the following correctly states the jacks conditions.

Calcium gluconate.

Accidental removal of one or both parathyroid glands can occur during a thyroidectomy. Which of the following is used to treat tetany.

No evidence of primary tumor.

According to the tumor node metastasis TNM classification system, T0 means there is:

Administer an antiemetic.

Adverse effects to chemotherapy are dealt with by patients and their caregivers every day. What would the nurse do to combat the most common adverse effects of chemotherapy?

Differentiating delirium from dementia.

An 80 year old man has been brought to the er by his daughter, who states that her father has become confused and agitated over the past several days, the daughter expresses fear that her father is getting senile and states that this concern prompt her to seek care, the er nurse and the other members of the care team should prioritize which of the following aspects of assessment.

Presbyopia

An aging client is brought to the clinic by the son. The son states that he has seen his parent holding reading materials at an increasing distance to focus properly. What age related changes does this indicate?

This medication helps significantly, but the benefits tend to decrease over time.

An older adult man has been diagnosed with Parkinson's disease and has begun treatment with levodopa and carbidopa when providing health education about his new medication regimen. What should the nurse teach the man?

Peripheral neuropathy

An older adult patient that has type 2 diabetes comes to the er, with second degree burns to the bottom of both feet and states, "i didn't feel too hot, but my feet must have been too close to the heater." What does the nurse understand is most likely the reason for the decrease in temperature sensation.

-best eye opening -best verbal response -best motor response

As part of a start of shift nursing assessment, the nurses documenting a patient's neurological status, according to the Glasgow Coma Scale, which reslonses will the nurse assess to determine the patients GCS score select all that apply.

Replace the IV solution.

Assessing a client at 4 PM a nurse notes, the 800 ml of normal saline solution has been infusing since it was hung at 4 PM yesterday. What would be the nurses next action?

IgG

At 39 weeks gestation, a pregnant client visits the physician for a scheduled prenatal checkup. The physician determines that the fetus has developed an infection in utero and sends the client to the er. The client is very concerned about the health of her unborn child. Based on knowledge of the immune system, the delivery room nurse explains about which immunoglobin that will be increased in the fetus at the time of birth and will be actively fighting the infection.

-abducens -facial

Austin holbritter a 3 year old male is being seen in the, er practice nursing. He is a victim of a MVA where he suffered blunt facial trauma, especially to the orbit of his skull, which of the cranial nerves are in danger of damage as a result of ocular injuries.

eustachian tubes

Austin holbritter a 6 month old male in his older brother Matthew, a 3 year old male, are being seen in the pediatric clinic where you practice nursing they are being seen by the physician for their third middle ear infection for this winter, the other reports they develop an upper respiratory infection and an ear infection seems quick to follow. What contributes to this event.

Abdominal distension

The nurse is caring for a client recovering from an esophagogastroduodenoscopy (EGD) which of the following client symptoms would require further nursing assessment.

Duodenum.

Cystic fibrosis, a genetic disorder characterized by pulmonary and pancreatic dysfunction, usually appears in young children but can also affect adults if the pancreas was functioning correctly, where would the bile and pancreatic enzymes enter the gi system?

bowel perforation

During a colonoscopy with moderate sedation, the patient groans with obvious discomfort and begins bleeding from the rectum, the patient is diaphoretic and has an increase in abdominal girth from distension. What complication of this procedure is the nurse aware may be occurring.

tinnitus and sensorineural hearing loss

During a pharmacology class, the students are told that some drugs need to be closely monitored. What aspect should the nurse closely monitor for in clients who have been administered salicylates, loop diuretics, quinidine, quinine & aminoglycosides

right atrium

Each chamber of the heart has a particular role in maintaining cellular oxygenation. Which chamber is responsible for receiving deoxygenated blood from the Venus system.?

Left atrium

Each chamber of the heart has a particular role in maintaining cellular oxygenation. Which chamber is responsible for receiving oxygenated blood from the lungs.?

Disrupting the heart during the critical period of ventricular repolarization.

Elective cardioversion is similar to defibrillation, except that the electrical stimulation waits to discharge until an r wave appears. What does this prevent?

Dopamine

Impaired balance and uncontrolled tremors of Parkinson's disease is correlated with which neurotransmitter.

Provide factual information and emotional support.

Family members of a client with traumatic brain injury are extremely distressed about their loved one. How can the nurse best assist the family to cope during this acute phase?

Means of transmission.

Flu and cold season offers excellent examples of psychologic reflexes to ward off illness. One problem is that an effective mechanical defense for one person can complete a link in the chain of infection for someone else. Which link is the above referring?

Electrocardiogram (ECG)

Following a myocardial infarction, a client develops an arrhythmia and requires a continuous infusion of lidocaine. To monitor the effectiveness of the intervention, the nurse should focus primarily on the clients:

Antiplatelets

Following a percutaneous transluminal coronary angioplasty (PTCA), which of the following medication classifications would be used to prevent thrombus in the stent.

it provides oxygen for energy and improves profusion to vital organs.

Having just witnessed her husband getting hit by a car, a woman does not feel well, she states my heart is beating so fast and my breathing is so fast to the nurse realizes this response to stress is normal. What is the purpose of this response?

anaphylaxis

Hey, patient is brought into the er by paramedics who report that when they arrived at the home, the patient's wife said 5 minutes prior to calling the patient had eating some chocolate covered peanuts. The wife went on to say the patient developed a nasal congestion, swelling sneezing, tearing of the eyes. By the time the paramedics arrived and began transport to the er, the patient complained of abdominal cramping and began having severe dyspnea, cyanosis, and hypotension. What is the patient diagnosed with?

Alcohol

Hey, post operative client calls to report he is experiencing a throbbing headache and flushed face after receiving cefotetan as an IV antibiotic prior to his surgery. What has the client consumed that has caused this reaction?

Aspirin

High doses of which medication can produce by lateral tinitus.

candidiasis

If you admit a client and discover on their head to toe assessment that they have thick cottage cheese like drainage coming from their vagina, you notice on their medication. Administration record that they have been taking oral antibiotics for the past 8 days due to an acute infection, is the most likely cause of the vaginal discharge.

stage IV it means that the cancer has spread to the other organs of the body.

If you are caring for a client who has just been told that they have stage IV on cancer the client asks you what stage IV means what would your best responce

stage IV

Sally ortiz is a 10 year old female is returning to the pediatric oncology group where you practice nursing sally has been diagnosed with cancer which has metastasized to her bones during client education, Sally's mom asked about tumor staging in its relation to Sally's condition. What stage would you expect Sally's tumor to be a sign?

an incompetent venous valve

Tenders explains to a patient that the primary cause of a varicose vein is:

mode of entry into the host.

The 6 elements necessary for infection include a causative organism, a reservoir of available organisms, a portal or mode of exit from the reservoir, a mode of transmission from reservoir to host a susceptible host and

Do not administer if respirations are less than 12 breaths per minute.

The client's physician prescribes opiate pain medication to manage post surgical pain which contraindication applies to opiate analgesics?

Benign tumors don't usually cause death.

The clinic nurses caring for a client who has just been diagnosed with a tumor of the client says to the nurse the doctor says my tumor is benign. What does that mean? What is the nurses best response?

sinoatrial node.

The electrical conduction system of the heart has several components, all of which are instrumental in maintaining polarization, depolarization and repolarization of cardiac tissue, which of the conductive structures is known as the pacemaker of the heart.

No one is to be touching the patient at the time the shock is delivered.

The emergency department nurse is caring for a patient who has gone into cardiac arrest, the nurses performing external defibrillation, which of the following is a vital step in the procedure.

mitral

The heart has valves between the atrium and the ventricles. What valve separates the left atrium and the ventricle?

Consistent use of mosquito repellents.

The infectious control nurses presenting a program on west Nile virus for a local community group to reduce the incidence of the disease the nurse should recommend. What action?

"Warfarin prevents clot formation in the atria of clients with atrial fibrillation."

The monitor technician on the telemetry unit as a charge nurse. Why every client whose monitor shows arterial fibrillation is receiving warfarin (coumadin). Which response by the charge nurse is best?

- ventilation - Gas exchange

What are the primary functions of the long select all that apply?

It targets normal body cells as well as cancer cells.

What disadvantages of chemotherapy should the patient be informed about prior to starting the regimen?

Fruits such as bananas and apricots.

What foods can the nurse recommend for the patient with hypokalemia?

It contracts and flattens.

What happened to the diaphragm during inspiration?

Friction rub.

What is a harsh, grating sound caused by abrasion of the pericardial surfaces during the cardiac cycle?

Dilated pupils

What is a symptomatic effect of the nervous system?

To decrease risk of transmission to vulnerable patients.

What is the best rational for health care providers receiving the influence of vaccination on a yearly basis?

Portal hypertension

What is the most common cause of esophageal varices?

-cardiac tamponade -pericardial effusion

a 46 year old man has been diagnosed with pericarditis and has begun treatment. When assessing this patient, the nurse should prioritize assessments relevant to what complications of pericarditis? Select, all that apply.

- death - permanent neurologic dysfunction - impaired cellular activity

a 58 year old construction worker fell from a 25 foot scaffolding and incurred a closed head injury as a result as his intracranial pressure continues to increase, the potential of herniation also increases if the brain herniates which of the following are potential consequences. Choose all correct options.

-age -cigarette smoking -occupation

a 67 year old is admitted for diagnostis studies to rule out cancer. The patient is caucasian, married and has been employed as a landscaper for 40 years and has a 36 year history of smoking a pack of cigarettes daily, what significant risk factors as the nurse recognize this patient has select all that apply.

Red open sores on the oral mucosa.

a client has been receiving chemotherapy to treat cancer, which assessment findings suggest that the client has developed stomatitis.

Ketoacidosis

a, 53 year old client is brought into the er via squad. Where you practice nursing, he is demonstrating fast, deep, labored breathing and has a fruity odor to his breath. He has a history of type one diabetes. What could be the cause of his current serious condition?

Morphine

the er nurse is caring for a patient with a suspected mi. What drugs should the nurse anticipate administering to the patient?

Pleural friction rub.

the nurse auscultates. The long sounds of a client during a routine assessment the sounds produce are harsh and cracking sounding like 2 pieces of leather being rubbed together, the nurse would be correct in documenting this finding as.

- it is substernal in location - it is sudden in onset and prolonged in duration. -it is viselike and radiates to the shoulders and arms

the nurse educating a patient diagnosed with angina pectoris. About the difference between the pain of angina and myocardial infarction. How should the nurse described the pain experienced during an mi select all that apply.

thirty seconds after administration, the facial weakness and ptosis will be relieved for approximately 5 min

the nurse is assisting with administering a tensolin test to a patient with ptosis. if the test is positive for myathenia gravis what out come does the nurse know will occur?

Mucus, acidic gastric secretions and saliva enzymes.

the nurse is caring for a client admitted to the acute care unit with a fever of unknown origin, considering how the immune system works, which of the following are considered chemical barriers to invading bacteria.

Sinus tachycardia.

A 26 year old client is returning for diagnostic follow up. Her halter monitor strip reveals a heart rate with normal conduction, but with a rate consistently above a 105 beats per minute, what type of dysrhythmia would you expect the cardiologist to diagnose?

Melatonin

A 30 year old client working arrives at the community clinic, complaining of difficulty sleeping which hormone should the nurse explain is responsible for regulating the sleep wake cycle.

Nausea and vomiting.

A 75 year old male patient presents at the er with symptoms of a small bowel obstruction in emergency room nurses obtaining assessment data from this patient. What assessment finding is characteristic of a small bowel obstruction?

Assess for a ruptured tympanic membrane.

A 78 year old client reports decreased hearing. In the right ear, the doctor orders irrigation of the right ear to remove cerumen which is the priority action by the nurse.

Arterial fibrillation.

A Perceptor is discussing stroke with a new nurse on the unit, the preceptor would tell the new nurse, which cardiac dysrhythmia is associated with cardiogenic embolic strokes.

peaked T waves

A client admitted to the telemetry unit has a serum potassium level of 6.6 mEq/L. Which electrocardiographic characteristic is commonly associated with this laboratory finding.

Arteriosclerosis is a loss of elasticity or hardening of the arteries that happens as we age.

A client asked the clinic nurse with the differences between arteriosclerosis and atherosclerosis.what is the nurses. Best response.

60

A client awaiting CABG for coronary artery diseases is amazed at the lack of symptoms, the nurse informs the client that symptoms usually do not manifest until an arterial lumen is out least____% occluded.

reducible

A client comes to the er. Because he's noticed a bulging in his groin area when he coughs, he states that he noticed this ever since he lifted a heavy bag of gardening mulch last week, the physician determines that the client has an Inguinal hernia and applies manual pressure to replace the protruding structures back into the abdominal cavity. The nurse understands that this procedure indicates that the hernia is which type.

Ineffective airway clearance related to bronchospasm and laryngeal edema.

A client comes to the, er, complaining of difficulty breathing. He states it feels like my throat is closing up, we were out to dinner and then I started feeling strange after I ate shrimp cocktail. The client is leaning forward with a respiratory rate of 36 breaths per minute. His skin is pale. The nurse suspects anaphylaxis, which nursing diagnosis would be the priority?

Cervical

A client complains of unusual vaginal discharge and heavier more painful periods than usual. This client symptoms may possibly be warning signs of which type of cancer.

Lung congestion

A client diagnosed with heart failure presents with a temperature of 99.1°F pose of 100 beats per minute respirations of 42 breaths per minute blood pressure is. 110/50mm Hg crackles in both lung bases, nausea and pull oximeter reading 89 percent, which finding indicates a need for immediate attention.

Droplet

A client diagnosed with influenza is admitted to the hospital. Which transmission based precautions should the nurse initiate.

-use of antibiotics -precautionary use of antibiotics with no infection -biologic adaption of microbes to interfere with antibiotics

A client has multi drug resistant sepsis. Several antibiotics have been ineffective in fighting the client subsets. What could cause this select all that apply.

The disease is spread by arthropods.

A client has received a diagnosis of lymes disease. What does the nurse understand about the transmission of infection resulting in this disease?

Incision of the eardrum.

A client has undergone a myringotomy. The nurse interprets this as which of the following.

pseudomembranous colitis

A client in the acute care facility for the administration of intravenous antibiotics to treat bacterial pneumonia, the client begins to have severe diarrhea 3 days after the ivy antibiotics with abdominal cramping and pain. What does the nurse suspect the client has developed due to the antibiotic use.

24 hours

A client is ordered. In an intravenous solution of ringers, lactate 1000 ml to infuse at 40 ml per hour, what is the maximum amount of time the nurse should allow this IV to hang.

Metabolic rate

A client is undergoing diagnostics for an alteration in thyroid function. What physiologic function is affected by altered thyroid function.

all options are correct

A client is undergoing sensitivity testing to define the allergen, which is causing the client difficulty. Before the procedure begins, the nurse indicates the various routes in which all allergen can be introduced. What is an allergen introduction route.

irrigate both eyes

A client presents the er reporting a chemical burn to both eyes. Which of the following nursing intervention is appropriate.

Diphenhydramine

A client presents to the clinic with complaints that he began to itch and break out in hives after taking an aspirin this morning. What medication Does the nurse anticipate ministering that blocks histamine receptors?

Chronic, traumatic encephalopathy

A client receives a diagnosis of concussion. While speaking with a client the nurse learns that this is the client's third head injury. This information is of particular significance because it puts the client at risk for.

The client is showing the fight or flight response.

A client who is frightened of needles has been told that he will have to have an IV line inserted the client's blood pressure and pulse rate increase in the nurse observes the pupils dilating. What does the nurse recognize has occurred with this client?

Calcium-channel blocker

A client with CAD thinks diltiazam (cardizem) has been causing nausea. Diltiazem (Cardizem) is categorized as which type of drug.

The aneurysm may be preparing to rupture.

A client with a diagnose abdominal aortic aneurysm develops severe low back pain, which is the most likely cause.

Preserving liver function.

A client with a lengthy history of alcohol addiction is diagnosed with cirrhosis. The nurse emphasizes that the principal goal of cirrhosis therapy is.

Check the client regarding chills, low back pain, dyspnea and skin itching during the transfusion.

A client with a plastic anemia is going to receive a blood transfusion in addition to taking vital signs and verifying that the unit of blood cells is matched to the client, what other assessments or actions would you take?

esophageal diverticula.

A client with an esophageal disorder comes to the hospital with symptoms that include halitosis and a sour taste in the mouth. These symptoms are associated most directly with which condition.

white blood cell(wbc) count 22.8/mm3

A client with complaints of right lower quadrant pain is admitted to the emergency department, blood specimens are drawn and sent to the laboratory, which laboratory findings should be reported to the physician immediately.

Serum glucose level of 52 mg/dl

A client with diabetes mellitus has a prescription of 5 units of U-100 regular insulin and 25 units of U-100 isophane insulin suspension (NPH) to be taken before breakfast at about 430 PM the client experiences headache sweating tremor, palLOr and nervousness. What is the most probable cause of these signs and symptoms?

2 to 6 weeks

A client with ineffective endocarditis is admitted to your unit. The client asked how long they will be on IV medication. What should you respond?

Barrel chest

A client with newly diagnosed emphysema is admitted to the medical surgical unit for evaluation, which does the nurse recognize as a deformity of the chest wall that occurs as a result of over inflammation of the lungs in this client population?

pallor

A client with raynauds disease complaints of cold and numbness in the fingers, which of the following would the nurse identify as an early sign of vasoconstruction.

cholelithasis

A client with the calculi in the gallbladder is said to have.

Thrombophlebitis.

A client's chart indicates Virchows triad. This client is at risk for which complication?

Disposing of sharps appropriately and not recapping needles.

A group of nurses have attached an inservice on the prevention of occupationally acquired diseases that affect health care providers. What action has the greatest potential to reduce a nurses risk of acquiring hepatitis c in the workplace?

Place the patient in a side lying position.

A hospital patient has experienced a seizure in the immediate recovery. What action best protects the patients? Safety.

"the pericardial surfaces lose their lubricating fluid because of inflammation and rub against each other."

A nurse and a nursing student are performing a physical assessment of a client with pericarditis, the client has an audible pericardial friction rub on auscultation. When leaving the room the student asked the nurse. What causes the sound? The nurses. Best response is which of the following.

The blood will seal the hole in the dura and prevent further losses to real spinal fluid.

A nurse and nursing students are caring for a client recovering from a lumbar puncture yesterday, the client reports. A headache despite being on bed rest overnight, the physician plans an epidural blood patch this morning, the student asked how this will help the headache the correct reply from the nurse is which of the following?

Infiltration

A nurse assessing the IV site of a patient observered swelling and pallor around the site and notes a significant decrease in the flow rate, the patient complains of coldness around the infusion site. What IV complication does this describe?

- bradycardia. -bradypnea - Hypertension.

A nurse caring for a patient with head trauma will be monitoring the patient for Cushing's triad, what will the nurse recognize as the symptoms associated with Cushing's triad? Select all that apply.

Occipital.

A nurse is completing a neurological assessment and determines that the client has significant visual deficits, considering the functions of the lobes of the brain. Which area will most likely contain the neurologic deficit.

-incubation period -prodromal stage -acute stage -convalescent stage -resolution

A nurse is explaining the steps of infection to a client. Place in the correct order the sequence the nurse would identify.

Gallbladder

A nurse is preparing a client for surgery during preoperative teaching. The client ask, where is bill store? The nurse knows that bile is stored in the:

Play distal tip to nose, then ear tip and end of the xiphoid process.

A nurse is preparing to place a patient's order. Nasogastric tube. How should the nurse best determine the correct length of the nasogastric tube?

Ptosis

A nurse notices that a clients left upper eyelid is drooping, the nurse has observed:

Appendicitis

A patient arrives in the emergency department with complaints of right lower abdominal pain that began 4 hours ago and is getting worse. The nurse assesses rebound tenderness at MC Burney's point. What does the assessment data indicate to the nurse?

3 minutes

A patient asks the nurse how long he will have to wait after taking nitroglycerin before experiencing pain relief. What is the best answer by the nurse?

Emotional and personality changes.

A patient brought to the clinic by his wife and son is diagnosed with Huntington's disease on providing anticipatory guidance. The nurse should address the future possibility of what effect of Huntington's disease.

a mucopurulent ocular discharge

A patient comes to the clinical they suspected eye infection. The nurse recognizes that the patient most likely has conjunctivitis as evidenced by what symptom.

irrigates the wound to remove debris.

A patient comes to the er. With a large scalp laceration after being struck in the head with a glass bottle after assessment of the patient, what does the nurse to be for the physician suture the wound.

To hearing loss will likely resolve what time after the drug is discontinued.

A patient diagnosed with arthritis has been taking aspirin and now reports experiencing tinnitus and hearing loss. What should the nurse teach this patient?

Severe headache

A patient is exhibiting classic signs of a hemorrhagic stroke. What complaint from the patient would be an indicator of this type of stroke?

To remove air from the pleural space.

A patient is exhibiting signs of pneumothorax. Following tracheostomy, the surgeon inserts a chest tube into the interior chest wall. What should the nurse tell the family as the primary purpose for this chest to?

Gingivitis

A patient is experiencing painful, inflamed and swollen gums, and when brushing the teeth, the gums bleed what common disease of the oral tissue does the nurse understand these symptoms indicate.

The tonsils help to guard the body from invasion of organisms.

A patient is having her tonsils removed the patient. Ask the nurse what function the tonsil serve. Which of the following would be most accurate response.

An intracerebral hematoma.

A patient sustained a head trauma in a diving accident and has a cerebral hemorrhage located within the brain. What type of hematoma is this classified as?

Older adults with compromised health status.

A public health nurse promoting the annual influence of vaccination is focusing health promotion efforts on the populations most vulnerable to death from influenza the nurse should focus on which of the following groups.

This must be a stressful time for you.

A recently widowed diabetic comments that her blood sugar levels are running higher than usual. Which is the best response from the nurse.

The pain occurs immediately following physical exertion.

A resident of a long term care facility has complained to the nurse of chest pain. What aspect of the residence pain would be most suggestive of angina as the cause?

Absence seizure

A school nurse is called to the playground where a 6 year old girl has been found unresponsive and staring into space, according to the playground supervisor. How would the nurse document the girls activity in her chart school?

Encourage fluid intake, if possible, to dilute the urine.

A side effect of chemotherapy is renal damage to prevent this. The nurse.

lacrimal

A young baseball player was fielding a ground ball when it bounced and struck the player in the left eye, leaving a large ecchymosis an edema and client education after applying an ice pack on nurse explains the functions of the various structures of the eye what glands contained in the eyelids produced tears.

- right mainstem Bronchus - Right upper lung.

A young client who aspirated a piece of candy has been brought to ER where would be the most likely places to find an aspirated piece of candy select all that apply?

Prophylaxis

After a client has had rheumatic fever as a child, they are always instructed to take an oral antibiotic before having dental cleanings or treatment. This is a case of which type of therapy.

Retinol detachment.

After a fall at home client hits their head on the corner of a table shortly after the accident, the client arrives at the er, unstable to see out of their left eye, the client tells the nurse at symptoms began with seeing spots or moving particles in their field of vision, but that there was no pain in the eye. The client is very upset that the vision will not return. What is most likely the cause of this client's symptoms.

-Diarrhea - Information of oral mucous membranes.

Bernie Thompson in 71 year old female with a history of a candidiasis is being closely monitored for signs of superinfection, which of the following signs and symptoms can indicate that superinfection has occurred. Choose all that apply.

Vertigo

But manifestation is most problematic for the client diagnosed with Meniere's disease.

Peritonitis

Diagnostic imaging and physical assessment have revealed that a patient with peptic ulcer disease has suffered a perforated ulcer. The nurse recognizes that emergency interventions must be performed as soon as possible in order to prevent the development. Of what complication.

Notify the neurosurgeon of the occurrence.

Following a spinal cord injury, a patient is placed in halo traction. While performing pin site care the nurse notes that one of the traction pins has become detached. The nurse would be correct and implementing what priority nursing action.

adjuvant therapy is likely

Following surgery for an adenocarcinoma, the client learns the tumor stages. T3 N1 M0 What treatment mode will the nurse anticipate?

heart failure

Following the morning assessment of an older adult patient, the nurses documented, "edema 3 + present to ankles and feet, dorsalis pedis and posterior tibial pulses palpable bilaterally." The nurse should recognize that this patient may be exhibiting symptoms:

Decreased cardiac output related to reduced myocardial contractility.

For a client with cardiomyopathy, the most important nursing diagnosis is:

the alveolar walls contain fewer capillaries

For which reason does gas exchange decrease in older adults?

maxillofacial injury and gurgling respirations

Four clients injured in an automobile accident, enter the er. At the same time the triage nurse evaluates them immediately, the nurse assign the highest priority to the client with the:

Maintaining intact skin.

In older adults with chronic conditions, there is a marked increase in risk for infection and excellent first line defense against infection in older adults is.

intermittent claudication

In the clinic you are caring for a client with Buergers disease. What is a common symptom of Buergers disease?

To decrease workload of the heart.

In the treatment of coronary artery disease, medications are often ordered to control blood pressure in the client, which of the following is a primary purpose of using beta-adrenergic blockers in the nursing management of CAD?

drink alcohol in moderation.

Kenneth London, a 52 year old male, is returning to follow up after his recent polysomnographic intended to rule out or confirm a suspected diagnosis of sleep apnea that pulmonologists reviewed the test findings and made recommendations to Mr. Lyndon to decrease the symptoms of his sleep apnea, which of the following is not a recommended lifestyle change for those diagnosed with severe sleep apnea.

Marshall has elongated eyeballs.

Marshall Dunbar, 7 year old second grader, is being seen by the pediatric opthamologist due to recent vision changes after his examination by the physician. You discuss diagnosis with his parents and explaining myopia. What do you indicate the cause of Marshall's vision change?

Type II cells.

Millions of alveoli for for most of the pulmonary mass. the squamous Epithelia cells lining each alveolus consists of different types of cells, which type of alveolar cells produce surfactant.

- stop the medication infusion at the first sign of extravasation. - Aspirate any residual drug from IV line - Administer an antidote, if indicated.

Nursing action for extravasation of a chemotherapeutic agent would include which of the following nursing actions. Select all that apply.

Time between exposure and onset of symptoms.

Nursing students are reviewing information about infectious diseases and events associated with infection students demonstrate understanding of the information when they identify the incubation. As which of the following.

acute

The client was hit in the head with a ball and knocked unconscious upon arrival at the er and subsequent diagnostic tests, it was determined that the client suffered from a subdural hematoma. The client is becoming increasingly symptomatic. How would the nurse expect this subdural hematoma to be classified?

Iodine

The most common type of goiter is caused by a lack of which of the following.

Maintain a clear airway to ensure adequate ventilation.

The most important nursing priority of treatment for a patient with an altered loc is to.

The potential for aspiration.

The most significant complication related to continuous to feeding is:

Atrial depolarization

The nurse cares for a client with dysrhythmia and understands that the p wave on an electrocardiogram represents which phase of the cardiac cycle.

The client needs intermittent IV medication administration.

The nurse caring for a client who has a medication lock on their being punctured advice knows that medication locks are used when

Begin cardiopulmonary resuscitation (CPR)

The nurse in the intensive care unit hears an alarm sound in the patient's room arriving in the room. The patient is unresponsive without a pulse and a flat line on the monitor. What is the first action by the nurse?

Pulse pressure

The nurse is assessing a patient's blood pressure. What does the nurse document as the difference between the systolic and the diastolic pressure?

-right ventricle -pulmonary artery -pulmonary vein -left atrium -aorta

The nurse is assessing the cardiovascular status of a client who was found unresponsive in a lobby area. Following transfer of the client, the family asked how blood circulates through the body. The nurse is most correct to state the proper circulation, as which? Place the pattern of circulation in the correct order beginning in the right atrium. Use all options.

nystagmus

The nurse is assisting with the administration of a caloric stimulation test. Which client response should the nurse document as an expected response.

From a break in the skin.

The nurse is caring for a client diagnosed with ineffective endocarditis and awaiting blood culture results, the client asks. Where did I pick up this bacteria? The nurses most safe to speculate. Which of the following.

Dorsiflexion of the foot noting calf pain.

The nurse is caring for a client following cardiac to file for placement, which nursing action is correct when obtaining a home in assigned to screen for thrombi phlebitis?

electroconvulsive therapy.

The nurse is caring for a client newly diagnosed with cancer, which of the following therapies is used to treat something other than cancer.

Assess the client.

The nurse is caring for a client on a monitored telemetry unit. During morning assessment, the nurse notes abnormal ECG waves on the telemetry monitor periodic action with the nurse do first.

ST elevation

The nurse is caring for a client presenting to the er reporting chest pain, which electrocardiograph ECG finding would be most concerning to the nurse.

Herniation occurs through the foramen magnum.

The nurse is caring for a client who continues to have increasingly high intracranial pressure, which complication is expected unless intracranial pressure is resolved.

Opportunistic

The nurse is caring for a client who has acquired immune deficiency disease aids and has developed oral thrush, what type of infection is the nurse aware that has developed due to immunocompromised state of the client?

an increase of concentrations of the tracer used in the scan.

The nurse is caring for a client who has had a nuclear scan to aid in the diagnosis of possible cancer. The scan showed a hot spot. What does this mean?

Irregular heart rate.

The nurse is caring for a client who is diagnosed with an infarction of the posterior wall of the right atrium. Which assessment findings would the nurse anticipate relating to the infarction location.?

The hair loss is usually temporary.

The nurse is caring for a client who is scheduled for chemotherapy, which is the best statement the nurse can make about the client experiencing chemotherapy induced alopecia

This is an accurate indicator of myocardial injury.

The nurse is caring for a patient admitted with unstable angina. The laboratory results for the initial troponin I is elevated in this patient. The nurse should recognize what implication of this assessment. Finding?

Intravenous diazepam (valium).

The nurse is caring for a patient on the neurologic unit who is in status epilepticus, what medication does the nurse know may be given to halt the seizure immediately?

Stop the test and monitor the patient closely.

The nurse is caring for a patient who is undergoing an exercise stress test. Prior to reaching the target heart rate, the patient develops chest pain. What is the nurses most appropriate response?

Rapid jerky and voluntary movements.

The nurse is caring for a patient with Huntington's disease in the long term care facility, what does the nurse recognizes the most prominent symptom of the disease that the patient exhibits?

The thin fibrous sac encasing the heart.

The nurse is caring for a patient with a diagnosis of pericarditis. where does the nurse understand the inflammation is located?

Achieve as high a level of function, as possible.

The nurse is caring for a patient with permanent neurologic impairments resulting from a traumatic head injury when working with this patient and family. What mutual goals should be prioritized?

Relieved by rest and nitroglycerin.

The nurse is part of a triage team that is assessing a patient to determine if his chest pain is a manifestation of angina pectoris or in mi, the nurse knows that a primary distinction is that the pain of angina is:

Pearly grey and translucent.

The nurse is performing an assessment of a patient's ear when looking at the tympanic membrane, the nurse observes a healthy membrane. What should the appearance be?

Wash your hands before and after installing eye drops and do not touch the tip of the bottle.

The nurse is teaching the client to instill eye drops. Which statement is correct.

- loosening constrictive clothing. - Positioning the patient on his or her side with head flexed forward. - Providing privacy

The nurses called to attend a patient who is having a seizure in the waiting area, what nursing care is provided for a patient who is experiencing a convulsive seizure. Select all that apply.

Severe back pain.

The nurses caring for a client with abdominal aortic aneurysm (AAA). Which assessment finding is most likely to indicate a dissection of the aneurysm.

-boost the immune system -increase white blood cell production

The nurses caring for a client with breast cancer who has been receiving chemotherapy. The client was admitted with an infected lesion on her left leg. The physician has ordered neupogen What will neupogen do for this? select all that apply.

Positive family history.

The nurses educating a patient at risk of atherosclerosis. What non modifiable risk factor does the nurse identify for the patient?

do you allow the infant to hold or prop the bottle during feeding?

The nurses in instructing the mother of an infant diagnosed with Otis media, the mother states, why is my child getting recurrent ear infections? Which assessment question is best?

An automatic external defibrillator.

The nurses in the mall and observes a clients slumped to the floor, the nurse assesses the client and notes no pulse the nurse calls for assistant to others in the mall and request. Which piece of equipment?

the sinoatrial node

The patient has a heart rate of 72 beats per minute with a regular rhythm. Where does the nurse determined the impulse arises from?

Reduced blood supply to the heart.

The physician orders medication to treat a client's cardiac ischemia. What is causing the client's condition?

Digestion of microbes by WBCs

The process of phagocytosis involves.

Obtaining a throat culture.

The school nurses providing care to a child with a sore throat with any sign of throat infection. The nurse stresses. Which of the following.

sawtooth

The staff educator is presenting a class on cardiac dysrhythmias. How would the educator described the characteristic pattern of the atrial waves in atrial flutter?

- need for careful monitoring for cardiac symptoms. - Need for carefully regulated exercise. - Need for dietary modifications

The student nurses is preparing a teaching plan for a patient being discharged status post MI. What should the student include in the teaching plan? Mark all that apply.

Liquid

The type of stool that will be expelled into the ostomy bag by a client who has undergone a surgery for an ileostomy will be:

- older adult receiving potassium chloride in the solution. - Middle aged adult receiving medication for congestive failure.

There are limited infusion pumps available on the nursing unit, which client has the greatest need for accurate fluid monitoring. Select all that apply.

To restore the flow of blood through the coronary arteries.

Thrombolytic therapy is being prepared for administration to an older adult patient who has presented to the emergency department with an ST segment elevation, mi. The nurse recognizes that the primary goal for this intervention is.

Enhanced myocardial oxygenation.

Thundering priority of care for a client exhibiting signs and symptoms of coronary artery disease should be to:

Mode of transmission.

Thunders places. A patient in isolation isolation techniques have the potential to break the chain of infection by interfering with what component of the chain of infection.

To dehydrate the brain and reduced cerebral

To dehydrate the brain and reduce cerebral edema. An osmotic diuretics, such as mannitol, is given to the patient with increased intracranial pressure ICP for which of the following actions.

-mycoplasmas -prions -viruses -fungi

Understanding the process of infection is integral to nursing practice infection is a disease process that results from which infectious agents select all that apply.

Amiodarone.

What is the drug of choice for a stable client with ventricular tachycardia.

It cushions the brain and spinal cord.

What is the function of cerebral spinal fluid CSF?

Regular

What is the only insulin that can be given intravenously.

purkinje fibers

What is the terminal point in the conduction system of the heart?

10%

What percentage of clients with a documented history of penicillin allergy are also allergic to cephalosporins?

Diabetes insipidus

What should the nurse expect when hourly assessment of urine output on a patient post craniotomy exhibits a urine output from a catheter of 1500 ml for 2 consecutive hours.

The client should consider getting a wig or cap prior to beginning treatment.

What should the nurse tell a client who is about to begin chemotherapy and is anxious about hair loss.

intrinsic factor

What term describes a gastric secretion that combines with vitamin b12 so that it can be absorbed?

A blood pressure of 176/88 mm Hg

When assessing a client with pheochromocytoma a tumor of the adrenal medulla that secretes excessive catecholamine the nurse is most likely to detect.

Third Degree

When no atrial impulse is conducted through the av node into the ventricles, the client is said to be experiencing which type of av block?

Assess the heart's mechanical activity.

When performing an ECG, It is also necessary to palpate a peripheral pulse in order to:

Tetracycline

Which antibiotic is associated with the permanent discoloration of tooth enamel?

Severe hypertension, slow heart rate, pounding headache and sweating.

Which are characteristics of autonomic dysreflexia.

Abdomen bruit

Which assessment? Finding by the nurses, the most significant findings suggestive of aortic aneurysm.

fibrinolytic

Which class of medication lyses and dissolves thrombi

My eyelids were stuck together this morning.

Which client statement would lead? The nurses suspect that the client is experiencing bacterial conjunctivitis.

92 year old female admitted for bilateral pneumonia.

Which client would be at risk for contracting MRSA?

Wide and pulse pressure.

Which finding indicates increasing intracranial pressure ICP and the client who has sustained a head injury.

Loss of balance.

Which finding is considered a positive finding of the Romberg test?

hemiplegia

Which is the most common motor dysfunction seen in clients diagnosed with stroke?

Rods

Which of the following structures are responsible for night vision?

Antigen

Which of the following terms describes foreign particles that enter a host and stimulate the body's immune response?

-Offer a diet with fruit juices and citrus fruits. - Monitor intake and output every shift.

Which of the following would be appropriate nursing interventions for a client with hypokalemia select all that apply.

Coughing

Which of the following would be considered a mechanical defense mechanism?

usually progressive and slow.

Which of the following would be consistent with a benign neoplasm?

Weight gain

Which of the following would be inconsistent as a common side effect of chemotherapy?

Atrial fibrillation

Which of the following, if left untreated can lead to an ischemic stroke.

Pancreas

Which organ is both an exocrine and endocrine gland?

Heart failure.

Which recurring condition most commonly occurs in clients with cardiomyopathy?

They gain access to the blood and lymphatic channels.

Which statement is true about malignant tumors?

Nose bleed

Which symptom may indicate thrombocytopenia?

Hypovolemia and fever

While assuming a client the nurse finds a heart rate of a 120 beats per minute, the nurse recalls that causes of sinus tachycardia include which of the following.

Cluster.

While making your initial rounds after coming on shift, you find a client thrashing about in bed complaining of a severe headache, the client tells you the pain is behind his right eye, which is red and tearing. What type of headache would you suspect the client is having?

Do not visit if you have had a recent infection.

You are a nurse working on a bone marrow transplant unit, your patient is scheduled to receive a bone marrow transplant. What information will you provide to the patients visitors?

cerebrovascular accidents.

You are admitting a client to the er brought in by the paramedics who has a presumptive diagnosis of emboli to the brain. What is the result of emboli to the brain?

Poor circulation.

You are assessing the skin of a client with a cardiovascular disorder and find sparse hair growth on the legs and thick toenails. What does this indicate?

Fungus.

a client visits the clinic with the complaint of a circular rash on the upper right arm, the rashes diagnosis tinea corporis what type of infection does the nurse anticipate the client will be treated for.

troponin

a client with severe angina, pectoris and electrocardiogram changes is seen by a physician in the emergency department in terms of serum testing is most important for the physician to order cardiac:

The manufacture and development of blood cells.

a clients family member ask what hematopoiesis is. What should the nurse tell the family member?

melatonin

a clients history reveals a disruption in his sleep wake cycle due to an increased exposure to light which hormone would the nurse suspect is being affected.

group A beta-hemolytic strep

a mother brings her 4 year old son into the pediatric clinic with a mild fever and a red, spotty rash that is beginning to fade, the child's heart rate is rapid and the rhythm is abnormal. The mother says the child has been healthy until about 3 weeks to go, and the child had a sore throat, the nurse suspects. Romatic carditis. What organism causes rheumatic carditis?

Diminished or absent breath sounds on the affected side.

the nurse is caring for a client who is scheduled for a lobotomy for a diagnosis of lung cancer while assessing with a subclavian vein, central line insertion, the nurses, the clients, oxygen saturation rapidly dropping the client complains of shortness of breath and becomes Tachypneic the nurse suspects a pneumothorax has developed further assistant assessment findings supporting the presence of a pneumothorax include what?

Confusion

the nurse is caring for a client with serum sodium concentration of 113new/L. The nurse should monitor the client for the development. Of which condition.

reddish hemorrhagic spots on the skin

the nurse is documenting assessment findings as a client is being admitted to a medical unit. Which of the following conditions present with a diagnosis of infectious endocarditis, is correct to be documented as petechiae?

A 58 year old caucasian woman with macular degeneration.

the nurse should recognize the greatest risk for the development of blindness. Is which of the following patients.

Administer fluorescein dye

while cleaning gutters. A client reports getting debris in the eye on inspection. No obvious foreign object is noted, which of the following diagnostic evaluation techniques would be most beneficial for this client.

Concussion

while. Snowboarding the patient fell and sustained a blow to the head. Resulting in a loss of consciousness, the client regained consciousness with an hour after arrival at the er was admitted for 24 hour observation is discharged without neurologic impairment. What would the nurse expect the client's diagnosis to be?

The importance of smoking cessation

you are caring for a client with Buerger's disease. You know that most of the care for this disease is carried out at home what would be important to teach the client with Buerger's disease?

Hypocalcemia

you are the surgical nurse caring for a 65 year old female patient who is post operative day one following a thyroidectomy during your shift assessment, the patient complains of tingling in her lips and fingers, she tells you that she has an intermittent spasm in her wrist in hand, and she exhibits increased muscle tone well. Electrolyte imbalance. Should you first suspect?

Dipyridamole (Persantine)

A patient is being scheduled for a stress test. The patient is unable to exercise during the test. The nurse would include information about which medication used for for pharmacologic stress testing.

Discontinue the transfusion.

A patient is receiving the first of 2 ordered units of packed red blood cells. Shortly after the initiation of the transfusion, the patient complains of chills and experiences a sharp increase in temperature. What is the nurses priority action?

The patient will remain in the clinic to be monitored for 30 minutes following the injection.

A patient with a severe environmental allergy is scheduled for an auto immune therapy injection. What should be included in teaching the patient about the treatment?

Prepare for pericardiocentesis.

A patient with pericarditis is experiencing cardiac tamponade. Which collaborative intervention should the nurse anticipate for this patient?

- client denies pain. - Right extremity pink - Right extremity comparable in size to left. - No bleeding or bruising noted.

The nurse is evaluating the expected outcomes following thrombolytic therapy for a right leg, deep vain thrombosis. Which of the following findings confirms a positive outcome? Mark select all that apply.

The risk for falls related to orthostatic hypotension.

The nurses assessment of an older adult client reveals the following data: lying BP 144/82mm Hg, sitting BP 121/69mm Hg, standing BP 98/56mm Hg. The nurse should consequently identify what nursing diagnosis and the patient's plan of care?

Splinter hemorrhage.

The nurses precepting a new graduate nurse. Together they are caring for a client with an ineffective endocarditis. What is a sign of ineffective endocarditisprecepting a new graduate nurse. Together, they are caring for a client with an ineffective endocarditis. What is a sign of ineffective endocarditis?

Orthostatic hypotension

The nurses providing discharge instructions to a client with unstable angina. The client is ordered. Nitrostat 1/150 every 5 minutes as needed for angina. Which side effect, emphasized by the nurse, is common, especially with the increased dosage?

Myocardial Ischemia

The physical therapist notifies the nurse that a patient with coronary artery disease (CAD) experiences a much greater than average increase in heart rate during physical therapy. The nurse recognizes that an increase in heart rate in a patient with CAD may result. In what?

An abscess

A client comes to the clinic and informs the nurse that he has a painful area under his armpit, the nurse observes a 2 centimeter raised area that is erythremic and has a white substance inside of it. What does the nurse suspect the client may be experiencing?

Mineral oil

A client comes to the emergency department reporting that a bee has flown into his ear and is stuck. The client reports a significant amount of pain which of the following would be most appropriate to use to remove the bee

Administer oxygen attach cardiac monitor, take vital signs and administer sublingual nitroglycerin.

A client in the emergency department claims of squeezing substernal pain that radiates to the left shoulder and jaw, he also complains of nausea and diaphoresis and shortness of breath. What should the nurse do?

-Atrial cell stimulation -AV node -bundle of His -bundle branches -purkinje fibers

A client is experiencing an irregular heartbeat. The client asks the nurse how a heartbeat occurs. The nurse explains the condition system of the heart beginning with the sinoatrial node (SA Node). Place the conduction sequence of the heart in order beginning with the SA node. As all options.

sputum culture

A client presents to the emergency and respiratory compromise. The client temperatures 102.4°F heart rate is 88 beats per minute and regular and blood pressure is 138/76 mm Hg. The client is dyspneic, pale and expectorating. Green. Tinged sputum the physician orders, medications including antibiotics, antipyretics, nebulizer treatments in IV fluids, a chest x ray in sputum culture are to be completed. Which physician order would the nurse to complete before beginning antibiotic therapy.

Nitroglycerin SL

A client presents to the er reporting anxiety and chest pain. After shoveling heavy snow that morning. The client says the nitroglycerin has not been taken for months, but upon experiencing this chest pain did take 3 nitroglycerin tablets. Although the pain has lessened, the client states " they did not work all that well." the client shows the nurse the nitroglycerin bottle and the prescription was filled 12 months ago. The nurse anticipates which order by the physician.

use semi-Fowler position during, and 60 minutes after an intermittent feeding.

A client receives tube feedings after an oral surgery. The nurse manages tube feeding to minimize the risk of aspiration, which measures should the nurse include in the care plan to reduce the risk of aspiration.

Avoiding using soap on the irritated areas.

A client receiving external radiation to the left thorax to treat lung cancer has a nursing diagnosis of risk for impaired skin integrity. Which intervention should be part of the client's care plan.

- slow the infusion rate. - Elevate the client's head. - Notify the physician.

A client receiving intravenous therapy is experiencing circulatory overload, which of the following nursing actions is correct. Select all that apply.

Shortness of breath.

A client reports a recent onset of chest pain that occurs sporadically with exertion. The client also has fatigue and mild ankle swelling, which is the most pronounced at the end of the day. The nurse suspects a cardiovascular disorder. What other client complaint increases the likelihood of a cardiovascular disorder?

Lower esophageal sphincter.

A client reports frequent episodes of reflex and describes regurgitating small amounts of stomach contents into the esophagus. What sphincter is most likely not performing its normal psychologic function.

Electrocardiogram

A client reports lightheadedness, chest pain and shortness of breath. The physician orders tests to ascertain what is causing the client's problems. Which test is used to identify cardiac rhythms?

Avoid caffeinated beverages.

A client tells the nurse " my heart is skipping beats again. I'm having palpitations." after completing a physical assessment, the nurse concludes the client is experiencing occasional premature atrial complexes. (PACs). The nurse should instruct the client to.

Carcinoma in situ, no abnormal regional lymph nodes and no evidence of distant metastasis.

A client undergoes a biopsy of a suspicious lesion. The biopsy report classified the lesion, according to the TNM staging system as follows: TIS N0 M0 what does this classification mean?

enucleation

A client was brought to the er via ambulance after being involved in a multiple car in MVA, in addition to several broken bones, a client sustained a penetrating eye injury, the er physician has just informed the clients family that the injury is caused inflammation in all 3 layers of the eye and the vitreous, and that is possible that this condition could lead to.

You may have eaten contaminated restaurant food.

A client who has just been diagnosed with hepatitis A ask how did I get this disease? What is the nurses best response?

Risk for injury.

A client who is disoriented and restless after sustaining a concussion during a car accident is admitted to the hospital, which nursing diagnosis takes the highest priority in the client's care plan.

O

A client with type. O blood and a history of anemia is scheduled for a blood transfusion to avoid a transfusion reaction. This client must receive which blood type.

sumatriptan succinate (imitrex)

A nurse is admitting a patient with a severe migraine headache and a history of acute coronary syndrome. What migraine medication would the nurse question for this patient?

Unequal response

A nurse is assessing a client who has been in a motor vehicle collision, the client directly and accurately answers questions. The nurse notes a contusion to the client's forehead, the client reports a headache. When assessing the clients pupils. What reaction would confirm increasing intracranial pressure?

Contact the physician and report her findings.

A nurse is assessing a client who underwent cardiac catherization finds the client lying flat on the bed his temperature is 99.8 ° F (37.7°C). His blood pressure is 104/68 mm Hg. His pulse rate is 76 beats per minute. She detects weak pulses in the leg distal to the puncture site. Skin on the leg is cool to touch. The puncture site is dry, but swollen. What is the most appropriate action for the nurse to take?

weight loss, nervousness and tachycardia.

A nurse is assessing a client with hyperthyroidism. Which findings should the nurse expect?

Dark red and moist.

A nurse is assessing the stoma of a patient with an ostomy. What would the nurses say is a normal healthy stoma?

Stimulation of the Vegas nerve.

A nurse is aware that the patient's heart rate is influenced by many factors. The nurse understands that the heart rate can be decreased by:

acute pain

A nurse is caring for a client immediately following an appendectomy, the nurse should assign which nursing diagnosis the highest priority.

Recognizing and promptly treating streptococcal infections

A nurse is presenting an educational event to the community, the nurses providing information on rheumatic heart disease. What would the nurse tell the audience is the most effective prevention of rheumatic heart disease?

- maintaining an upright position following meals. - Avoiding foods that intensify symptoms.

A nurse is providing education to a client with gerd. The client asked what measures can be taken independently to help reduce the symptoms. Which interventions would the nurse recommend select all that apply.

- wash hands thoroughly. - Wipe the lids and lashes in a direction away from the nose with a moistened soft gauze pad, paper tissue or cotton ball. - Pull the tissue near the cheek downward, forming a sac in the lower lid. - Tilt, the head slightly backward and toward the eye in which the medication is to be instilled. - instill, the prescribed number of drops into the conjunctival pocket or apply a thin ribbon of ointment directly into the conjunctival pocket, beginning at the inner corner and moving outward. - Close eye gently.

A nurse is teaching a client instructions on how to instill I medications at home place, the steps the client should follow in the correct sequence from first to last.

- the client does not want to be social. - The client responds inappropriately. - The client nodes the head and smiles. - The client withdraws from activity.

A nurse is working in a long term care facility, which clues that the nurse notes suggest that the client is not hearing what the nurse said select all that apply.

remove the triggering stimulus.

A nurse on the neuro ICU is mentoring a new graduate working on the unit today they are talking about autonomic dysreflexia. What would the mentoring nurse tell the new graduate are the objectives of the care provided a patient experiencing autonomic dysreflexia.

20/200

A nurse practitioner examines a patient in documents a best corrected visual acuity, BC VA ratio in his is better eye that qualifies him for government financial assistance based on the definition of legal blindness is the ratio.

Pitting edema

A nurse presses fingers into a client's skin and quickly draws the fingers away. The nurse notes that indentation remains. What term would the nurse used to describe the finding?

A 39 year old man with chronic alcoholism.

A nurse who provides care in a walk in clinic assesses a wide range of individuals the nurse should identify which of the following patients is having the highest risk for chronic pancreatitis.

When hands are visibly soiled from client care.

A nurse would perform hand washing instead of using an alcohol based product. For which situation.

When the pouch is 1third to 1 half full.

A nurses caring for a client with an ostomy pouch, when should the nurse asked the client to empty the pouch?

Beta cells of the pancreas.

A nurses explaining the action of insulin to a client with diabetes mellitus during client teaching the nursery views the process of insulin secretion in the body, the nurse is correct when she states that insulin is secreted from the:

-Wheezes -Wet lung sounds

A nurses is conducting procedures to determine the extent of a client's left sided heart failure.. What adventitious lung sounds would the nurse expect to hear during ausculation of the lungs to support the diagnosis? Select all that apply.

-aortic -mitral -tricuspid -pulmonic

A nursing student is taking a test on the cardiovascular system, which of the following with a student correctly identifies cardiac valves. Select all to apply.

complete heart block

A pacemaker is the treatment of choice for what cardiac dysrhythmia?

Prevent a stroke by removing atherosclerotic plaque blocking cerebral flow.

A patient diagnosed with transient ischemic attack, tia is scheduled for a carotid endarterectomy. The nurse explains that this procedure will be done to what.

- assessing the peripheral pulses in the affected extremity. - Checking the insertion site for hematoma formation. - Elevating temperature and color in the affected extremity.

A patient had a cardiac catherization and is now in the recovery area. What nursing intervention should be included in the plan of care? Select all that apply.

-forced fluids (unless contraindicated) -get plenty of bed rest -take some over the counter analgesics

A patient had a lumbar puncture 3 days ago at the outpatient clinic and calls the nurse with complaints of throbbing headache. What can the nurse educate the patient to do for relief of the discomfort? Select all that apply.

Myocardium

A patient has a myocardial infarction and has been diagnosed as having damage to the layer of the heart responsible for the pumping action. The nurses are aware that the damage occurred. Where?

Insertion of a pacemaker

A patient has a persistent third degree heart block and has had several periods of syncope. What priority treatment should the nurse anticipate for this patient?

" that can be caused by ingestion of strong acids." " You may have ingested some irritating foods." " Is it possible that you are over using aspirin?"

A patient has been diagnosed with acute gastritis and asked the nurse what could have caused it. What is the best response by the nurse? Select all that apply.

IV Hyromorphone (Dilaudid)

A patient has been diagnosed with acute pancreatitis, the nurses addressing the diagnosis of acute pain related to pancreatitis what pharmacologic intervention is most likely to be ordered for this patient.

" cardioversion will essentially rest the cells in your heart that control the electrical activity."

A patient has been scheduled for cardioversion in the treatment of a tackyarrythmia but is unclear about the particulars of the procedure after signing the necessary informed consent. How can the nurse best explain this procedure to the patient?

nystatin (myocostatin)

A patient has been taking a 10 day course of antibiotics for pneumonia. The patient has been having white patches that look like milk curds in the mouth. What treatment will the nurse educate the patient about?

Demonstration

A patient has received a temporary ostomy during her treatment for colon cancer, which of the following techniques is most likely to facilitate the patient's ability to independently empty and change the ostomy after discharge.

Risk for falls.

A patient has suffered cerebral trauma after falling off of a ladder. The patient has been stabilized and is now receiving care on a neurological unit on planning this patient's care, what nursing diagnosis is most likely to result from an injury to this part of the brain.

Basilar

A patient is admitted to the emergency room. With a fractured skull sustained in a motorcycle accident, the nurse notes fluid leaking from the patient's ears. The nurse knows that this is a probable sign of which type of skull fracture.

- severe eye pain - Reddening of the eye. - Sudden onset of visual disturbance. - Nausea and vomiting.

A patient presents to an eye clinic with a number of symptoms related to his diminished vision and initial history leads the nurse practitioner to suspect that the patient has acute angle closure glauoma, which of the following symptoms would apply to the diagnosis. Let all that apply.

Achalasia

A patient tells the nurse that it feels like food is sticking in the lower portion of the esophagus. What motility disorder does the nurse suspect these symptoms indicate.

Infection of the knee.

A patient trips. While ambulating and breaks open the skin on his knee the next day, then he is warm red to touch and painful at the sight of injury. The patient CBC shows a high white blood count. What would the nurse suspect is wrong with the patient?

Ventricle fibrillation

A patient who has a myocardial infarction is experiencing severe chest pain and alerts the nurse. The nurse begins the assessment, but suddenly the patient becomes unresponsive, no pulse with the monitor showing a rapid, disorganized ventricle rhythm. What does the nurse interpret this rhythm to be?

Brain stem

A patient who has suffered a stroke is unable to maintain respiration is intubated and placed on mechanical ventilator support. What portion of the brain is most likely responsible for the inability to breathe?

Benefits of levodopa-carbidopa often diminish after one or 2 years of treatment.

A patient who is diagnosed with Parkinson's disease several months ago recently began treatment with levodopa -carbidopa, the patient and his family are excited that he has experienced significant symptom relief. The nurse should be aware of what implication of the patient's medication regimen.

A person whose vision is 20/40 can see an object from 20 feet away that a person with 20/20 person can see from 40 feet away.

A patient who presents for an eye examination is diagnosis having a visual acuity of 20./40 the patient asked for the nurse what these numbers specifically mean. What is a correct response by the nurse.

myxedema coma

A patient with a history of hypothyroidism is admitted to the intensive care unit unconscious with a temperature of 95.2°F. A family member informs the nurse at the patient has not taken thyroid medication and over 2 months. What does the nurse suspect that these findings indicate?

Antibiotics have no effect on viruses.

A patient with an upper respiratory infection, also known as a common cold, tells the nurse I am so angry with the nurse practitioner because he would not give me any antibiotics, what would be the most accurate response by the nurse?

Throbbing headache or dizziness

A patient with angina is beginning nitroglycerin before administering the drug. The nurse informs the patient that immediately after administration the patient may experience what.

Confusion

A patient with generalized seizure disorder has just had a seizure the nurse would assess for what characteristic associated with the postictal state.

Meningitis.

A patient with increased intracranial pressure. ICP has a ventriculostomy for monitoring their ICP. The patient is now exhibiting nuchal rigidity and photophobia. What would the nurse be correct in suspecting has become a complication.

Lymphatic circulation

A patient's most recent diagnostic imaging has revealed that his lung cancer has metastasized to his bones and liver, what is the most likely mechanism by which the patient's cancer cells to spread.

Decompression

A preoperative client scheduled to have an open cholecystectomy says to the nurse "the doctor said that after surgery I will have a tube in my nose that goes into my stomach, why do I need that," what most common reason for a client having a nasogastric tube in place after abdominal surgery should the nurse include in a response?

Risk for infection.

After being seen in the oncology clinic, a client with severe bone marrow suppression is admitted to the hospital, the clients cancer therapy consists of radiation and chemotherapy when developing the care plan for this client. Which nursing diagnosis takes priority.

Serum potassium level.

After being sick for 3 days, a client with a history of diabetes is admitted to the hospital with diabetic ketoacidosis. The nurse should evaluate which diagnostic test results to prevent dysrhythmias.

- poor, abstract reasoning - Decreased attention span. - Short and long term memory loss.

After having a stroke, a patient has cognitive deficits. What are the competive deficits? The nurse recognizes the patient as a result of the stroke. Select all that apply.

Defibrillation

After observing a code blue situation a nursing student asking a member of the code team with the treatment of choices for witness ventricular fibrillation. The best response by the nurses. Which of the following?

Ventricular depolarization

During electrical cardioversion, the defibrillator is set to synchronize with the electrocardiogram so that the electrical impulses discharges during:

exophthalmos

During the physical examination of a client with a suspected endocrine disorder, the nurse observes an abnormal bulging of the eyes. The nurse documents this finding as which of the following.

decreased cardiac output

The nurse is caring for a client who is displaying a third degree av block on the EKG monitor the client is symptomatic due to the slow heart rate, the most appropriate nursing diagnosis for this client would be. Which of the following.

A puncture at the radio artery.

The nurse is caring for a client who is in respiratory distress. The physician orders it arterial blood gas abg to determine various factors related to blood oxygenation. What site can abg be obtained from?

Every 72 hours.

The nurse is caring for a client with an intravenous infusion. she is looking up there institutions policy on changing IV equipment used in a venipuncture. When is most IV tubing changed?

Glucagon

The nurse is caring for a client with diabetes who developed hypoglycemia. What can the nurse administer to the client to raise the blood sugar level?

positive chvosteks sign

The nurse is caring for a client with hypoparathyroidism. When the nurse taps the client's facial nerve, the client's mouth twitches and the jaw tightens. What is their response documented as related to the low calcium levels?

"you will be vulnerable to an effective endocarditis for the rest of your life.

The nurse is caring for a client with ineffective endocarditis. The nurse teaches the client that they will have to take antibiotics periodically for the rest of their life the client wants to know why. What would be the nurses best answer to the client?

One part of the intestine telescopes into another portion of the intestine.

The nurse is caring for a client with intussusception of the bowel. What does the nurse understand occurs with this disorder?

Meningitis

The nurse is caring for a client with recurrent ear infections. The nurse assesses the client for further infectious process, traveling deeper into the tissue and becoming more lethal. Which infection originated in the ear is of most concern.

An arterial blood gas abg study

The nurse is caring for a patient admitted with acute exacerbation of chronic obstructive pulmonary disease during assessment the nurse finds that this patient is experiencing increased Dyspnea what is the most accurate measurement of the concentration of oxygen in the patient's blood.

Drawing venous blood to perform a blood patch.

The nurse is caring for a post lumbar puncture client who is experiencing an intense headache if the physician chooses aggressive treatment, which nursing action is anticipated.

- always face the client when talking. - Provide rate instructions and information.

The nurse is caring for an 86 year old client with hearing impairment, the nurses preparing to educate the client on the diagnosis and discharge plan. What action should the nurse take when talking with the clients select all that apply.

ineffective cardio pulmonary tissue perfusion secondary to coronary artery disease CAD.

The nurse is caring for an adult patient who had symptoms of unstable angina during admission to the hospital, the most appropriate nursing diagnosis for the discomfort associated with angina. Is what?

dextrose water is nutritionally and calorically inadequate.

The nurse is carrying for a client who has been on simple IV solutions for 2 days. Why should the nurse avoid maintaining clients on simple iv solutions such as dextrose and water longer than 1 to 2 days?

brachytherapy.

The nurse is completing an admission assessment for a client receiving interstitial implants for prostate cancer than our documents. This as.

" increased LDL and decreased HDL increase my risk of coronary artery disease."

The nurse is conducting patient teaching about cholesterol levels. When discussing the patients elevated LDL and lower HDL levels, the patient shows an understanding of the significance of these levels by stating what?

Improved cardiac output.

The nurse is creating a plan of care for a patient with cardiomyopathy. What priority go should underline most of the assessments and interventions that are selected for this patient.

Follicle stimulating hormone (FSH)

The nurse is educating a couple who has had difficulty with conception, the client ask about the hormone which is responsible for the production of eggs. The nursing answers that this hormone is called:

- sloughing tissue - tissue necrosis - effectiveness of the antidote

The nurse is preparing to assess a client whose chart documents the client experience extravasation on receiving the vesicant vincristine during the previous shift, the documentation also notes that an antidote is administered immediately, the nurse prepares to assess for which condition. Select all that apply.

black

The nurse is to obtain a stool specimen from a client who reported that he is taking iron supplements. The nurse would expect the stool to be with color.

Establish falls prevention measures.

The nurse providing care for a patient with cushings syndrome has identified the nursing diagnosis of risk for injury related to weakness. How should the nurse best reduce this risk?

ophthalmoscope

The nurse recognizes that the tool used to examine the inside of the eyes and shown in the accompanying images called the.

-Bladder distension -poikilothermic - No perspiration below the level of injury.

The nurse suspects that a newly admitted client is in spinal shock. What are the symptoms of spinal shock? Select all that apply.

10

The nurse teaches the client that a tetanus vaccination is recommended every____ years.

Extraocular muscle function.

The nurse to ask a client to follow the movement of a pencil up down, right, left in both ways diagonally, the nurses assessing which of the following.

Notify the physician.

The nurse working in the recovery room is caring for a client who had a radical neck dissection the nurse notices that the client makes a course, high pitched sound upon inspiration. Which intervention by the nurse is appropriate?

Monitor the patient closely to prevent infection.

The nurse working on a bone marrow unit knows that it is priority to monitor which of the following and a client who has just undergone a bone marrow transplant.

ecchymosis over the mastoid

The nurse working on the neurological unit is caring for a client with a basilar skull fracture during assessments and nurse expects to observe battle sign, which is a sign of basilar skull fracture, which of the following correctly describes battle sign.

Lopressor

The nurses administering a beta blocker to a patient in order to decrease automaticity. Which medication will the nurse administer?

Decreases the sinoatrial node. Automaticity.

The nurses administering a calcium channel blocker to a patient who is systematic sinus tachycardia at a rate of 132 BPM. What is the anticipated action of the drug for this patient?

Sodium level

The nurses administering furosemide to promote urinary excretion of excessive fluids for a client with cirrhosis when administering furosemide to this client. What should the nurse closely monitor?

lactulose

The nurses administering medication to a client that has elevated ammonia due to cirrhosis of the liver. What medication will the nurse give to detoxify ammonia and to act as an osmotic agent?

Respirations are increasing.

The nurses analyzing a client's blood Ph of 7.1, which symptom would indicate that the client's body is working to stabilize.

Document that the chest drainage system is operating as it is intended.

The nurses assessing a patient who has a chest tube in place for the treatment of a pneumothorax, the nurse observes that the water level in the water seal rises and falls in rhythm with the patient's respiration. How should the nurse respond to this finding?

Bone marrow aspiration.

The nurses assisting the physician with obtaining a sample to determine the status of blood cell formation. What type of procedure will the nurse have prepared the client for?

Sinus bradycardia

The nurses assisting with the monitoring of a client with a dysrhythmia that shows the pattern in the accompanying image. What does ram does the client have?

cardiac glycosides

The nurses caring for a client with cardiomyopathy. What drugs should the nurse expect to be ordered to promote effective heart contractions and adequate cardiac output?

" cardiac catherization is usually done to assess how blocked or open a patient's coronary arteries are."

The nurses caring for a patient admitted with angina who is scheduled for cardiac catherization. The patient is anxious and asked the reason for this test. What is the best response?

Increase in the size of the arteries lumen

The nurses caring for a patient who has undergone percutaneous transluminal coronary angioplasty (PTCA). What is the major indicator of the success for this procedure?

Blood gases

The nurses caring for an adolescent client in Jordan, a boarding accident, the client has a head injury, a fractured right rib in various abrasions and contusions, the as a blood pressure of 142/88 mmhg, pulse of 102 beats per minute and respirations of 26 breaths per minute, which laboratory tests best provides data on a potential impairment in ventilation.

" an aspirin a day can help prevent some of the blockages that can cause chest pain or a heart attack."

The nurses doing discharge teaching with a patient who has coronary artery disease. The patient asks why he has to take an aspirin every day if he doesn't have any pain. What would be the nurses best response?

20 feet distance.

The nurses establishing a visual test using the snellen charter, a client experiencing visual changes at which distance should the nurse instructs the client to stand.

-efficiency of heart as a pump -adequacy of circulating blood volume -patency and responsiveness of the blood vessels

The nurses evaluating a patient's diagnosis of arterial insufficiency with reference to adequacy of patient's blood flow. On what physiological variables does adequate blood flow depend. Select all that apply.

Iodine allergy

The nurses in the radiology unit of the hospital. The nurse is caring for a client who is scheduled for a lung scan, the nurse knows that the lung scan is the use of radioisotopes and a scanning machine before the perfusion scan. What must the client be assessed for?

- the patient was diagnosed as Sensorineural hearing loss. - The patients hearing did not improve appreciably with the use of hearing aids.

The nurses providing care for a patient who is just benefited from a cochlear implant, the nurse should understand that this is patients, health history likely includes which of the following select all but apply.

low LDL values and high HDL values

The nurses providing care for a patient with high cholesterol and triglyceride values and teaching the patient about therapeutic lifestyle changes such as diet and exercise the nurse realizes that the desired girl for cholesterol levels is which of the following:

Places. The client in a private room.

The nurses receiving a client with a radioactive implant for the treatment of cervical cancer. The nurse.

Gigantism

The nurses reviewing a client's history, which reveals that the client has had an over secretion of growth hormone that occurred before puberty. The nurse interprets this as which of the following.

- previous history of lung disease in the patient or family. - Occupational and environmental influences. - Previous history of smoking

The nurses taking a respiratory history for a patient who has come into the clinic with a chronic cough what information should the nurse obtained from the patient select all that apply

colloid solutions

The nursing instructor is discussing the different types of IV fluids with the nursing students, what type of fluid would the instructor tell the student is used to replace circulating blood.

Prophylactic

The oncologists advises a client with an extensive family history of breast cancer to consider a masectomy. This procedure would be considered. Which type of surgery.

-sores that don't heal -unusual bleeding or discharge -persistent indigestion

The public health nurses giving a talk on the warning signals of cancer to a local community group, a which of the following are the warning signals of cancer. Select all that apply.

state her role and name immediately after entering the patient's room.

The registered nurse taking shift report learns that an assigned a patient is blind. How should the nurse best communicate with this patient?

1.5-2.5 times normal

When a client has been diagnosed with DVT, they are typically placed on oral anticoagulant therapy to decrease the likelihood of developing additional thrombi. Warfarin is a trial and error pursuit with frequent testing. What PTT and aPTT levels are desired to achieve optimum drug effects.

use short, simple sentences.

When communicating with a client who has sensory (receptive) aphasia, the nurse should.

The secretions are released directly into the bloodstream.

When describing the difference between endocrine and exocrine glands, which of the following would the instructor include as characteristic of endocrine glands?

Increases the heart's oxygen demands.

When discussing angina pectoris secondary to atherosclerotic disease with the patient, the patient asked why he experiences chest pain with exertion, the nurse in forms the patient. That exertion:

During diastole

When do coronary arteries primarily receive blood flow?

Within 4 to 10 days after starting the antibiotics.

Which is the most likely time frame for pseudomembranous colitis to occur in after a patient has begun treatment antibiotics.

Maintain client on bedrest.

Which nursing intervention should a nurse perform to reduce cardiac workload in a client diagnosed with myocarditis?

Place earplugs into the ears before swimming.

Which nursing suggestion would be most helpful to the client with recurrent Otitis externa?

Hypothalamus

Which of the following areas of the brain are responsible for temperature regulation?

Weakness, diaphoresis, diarrhea 90 minutes after eating.

Which of the following assessment findings would be most important for indicating dumping syndrome in a post gastrectomy client.

A client with colitis and bloody diarrhea.

Which of the following clients would be a candidate for total parenteral nutrition?

amylase

Which of the following digestive enzymes, aids in the digesting of starch?

candidiasis

Which of the following disorders is a superinfection?

sulfamethoxazole/trimethoprim (bactrim) PO

Which of the following drugs is often used to treat an uncomplicated urinary tract infection?

Sudden onset of swelling of the lips, tongue and airway.

Which of the following is a symptom of anaphylaxis after IV antibiotic administration?

Phagocytosis

Which of the following is a term used to describe the process of ingestion and digestion of bacteria by cells?

Bradycardia

Which of the following is an inaccurate clinical manifestation associated with hemorrhage?

-Experiences a prolonged bleeding from an obvious injury. -Has unexplained blood loss as in rectal bleeding nosebleeds, bleeding gums or vomiting blood. -Feels fatigued with normal activities.

Which of the following reports from the client during the health history would lead the nurse to suspect that the client has a disorder of hematopoietic or lymphatic system? select all that apply:

Isotonic

Which of the following solutions is a crystalloid solution that has the same osmotic pressure as that found within the cells of the body and is used to expand the intravascular volume.

Deviation from the midline.

While conducting the physical examination during assessment of the respiratory system, which conditions does the nurses say by inspecting and palpitating the Trachea?

Persistent hoarseness is a warning sign of cancer.

You are admitting a client to your unit. While assessing the client you find out, the client has hoaerseness for approximately 4 months now why would you report this to the attending physician?

Arteriolar vasoconstriction

You are caring for a patient who is diagnosed with raynauds disease. What is the disease characterized by?

Intravenous morphine sulfate.

You are caring for a patient who just returned from the post anesthesia care unit, pacu and rates current pain as 9 out of 10, which of the following prescribed medications will provide the fastest relief from pain.

Enzymes

You are monitoring. The results of laboratory tests performed on a client admitted to the cardiac ICU with a diagnosis of myocardial infarction. Which test would you expect to show elevated levels?

Then use the blood and 4 hours or less after refrigerating it.

You are precepting a senior nursing student for the day one of your clients needs a blood transfusion. What would you explain to the student about interventions a nurse should perform to minimize the risk for aseptic reaction due to bacterial contamination of blood used for transfusion.

-debilitated clients -older adults -clients with impaired skin

You are teaching a pathophysiology class 2 pre nursing students today you are teaching about infection. What group of people would you tell the students are at increased risk for infection? Mark all that apply.

The infection is being transmitted by health care personnel.

You work on a long term care unit. In the last 2 weeks, more than half the clients on your unit have been diagnosed with gastroenteritis. Is the most likely reason.

Direct contact

a client arrives at the clinic with the complaint that she is having a vaginal discharge after having sexual intercourse with her boyfriend one week ago, the client is diagnosed with gonorrhea and given a prescription for treatment. What type of infection transmission does this nurse understand occurred?

the pons

a client experiences a head injury in a motor vehicle accident. The client's level of consciousness is declining, and respirations have become slow and shallow when monitoring a client's respiratory status, which area of the brain would the nurse realizes is responsible for the rate and depth.

y- administration tubing

a client has just had a transfusion ordered for severe anemia. You are gathering the supplies that you need in order to transfuse the client. What kind of tubing do you know that you need to infuse blood or blood products?

eyeballs that are shorter than normal?

a client has received a diagnosis of hyperopia and is wondering if there is a physical condition that has caused these vision changes in explaining hyperopia. What does the nurse indicate? Is the cause of the client's vision changes.

Flush the lock with saline or heparinized saline

a client is going out on path for the afternoon with their family, the physician has ordered that there being a puncture device needs to be temporarily capped. How will you ensure that the vein remains patent?

Use a soft toothbrush and avoid electronic toothbrush.

a client is receiving radiation therapy. The client ask you about oral hygiene. We advise regarding oral hygiene. Should you offer?

an allergy

a client presents at the walk in clinic with watery eyes, sneezing, itching and redness in the conjunctiva bilaterally. What might the nurse suspect the client is suffering from?

universal recipient

a nurse in the ER is providing care for a trauma client following a MVA. the clients blood donor card indicates type AB blood. people with type AB are:

Disorientation and restlessness.

a nurse instructor is discussing increased intracranial pressure ICP with a senior nursing student. What would the instructor be correct in telling the student is an early clinical manifestation of ICP.

Early diagnosis and treatment of gastroesophageal reflux disease.

a nurse is addressing the prevention of esophageal cancer in response to a question posed by a participant in a health promotion workshop. What action has the greatest potential to prevent esophageal cancer?

-unequal pupils -pinpoint pupils -absence of pupillary response

a nurse is assessing the client's pupils following a sports injury, which assessment findings indicate a neurologic concern. Select all that apply.

-hyperlipidemia -obesity -tabacco use

a nurse is educating a community group about coronary artery disease. One member asked about how to avoid coronary artery disease which of the following items are considered a modifiable risk factors for coronary artery disease. Choose all that apply.

On the cheeks below the eyes.

a nurse practitioner diagnosed a patient with an infection in the maxillary sinuses, select the area that the nurse palpated to make that diagnosis.

-pulsatile abdominal mass -low back pain -lower abdominal pai

a nurse suspects the presence of an abdominal aortic aneurysm. What assessment data with the nurse correlates with a diagnosis of abdominal aortic aneurysm? select all that apply.

Giardia lamblia

a patient is admitted with severe dehydration related to diarrhea. The patient was hiking in the mountains during a camping trip and drank water from a mountain stream without purifying it, what does the nurse know is the most likely cause of this diarrhea?

epidural hematoma

a patient is brought to the hospital after a skiing accident was unconscious for a brief period of time at the scene, then woke up disoriented and refused to go to the hospital. The patient became very agitated and restless, then quickly lost consciousness again. What type of TBI is suspected in the situation?

Cardiac and respiratory status

a patient who just suffered a hemorrhagic stroke is brought in to the ER, what would the nurses primary assessment focus on?

echocardiography

a patients gradual decline in activity tolerance and increased shortness of breath have promoted her health care provider to assess the structure and size of her heart. Which of the following diagnostic tests is most likely to yield these assessment data.?

The client may have cardiac or renal disorder.

a physician orders. IV solution for a 92 year old client to treat dehydration. The order was written to administer the fluid at a slower rate than what the nurse expected. Why would the physician designate the slower infusion rate for this client?

The cells are denying adequate oxygen because most of the oxygen in the body is transported by hemoglobin and red blood cells.

a student nurses caring for a client who is severely anemic than struct or ask the student how anemia affects the transport of oxygen to the cells. What would be the students best answer?

Level of consciousness.

the nurses reviewing the trend of a patient. Scores on the Glasgow Coma Scale. This allows the nurse to gauge what aspect of the patient's status.


Related study sets

Real Estate - Level 5, Chapter 3 - Taxes and Real Estate

View Set

Outline the origins of common law

View Set

operation management pt.2 production system is characterized by

View Set

Ch. 1 Review Questions (Nutrition)

View Set